You are on page 1of 15

Problem

Let , where . Determine


the minimum value taken by for in the interval .
Solution
It is best to get rid of the absolute value first.
Under the given circumstances, we notice that , ,
and .
Adding these together, we find that the sum is equal to , of which the minimum
value is attained when .
Edit: can equal or (for example,
if and , ). Thus, our two "cases"
are (if ) and (if ). However, both of these cases
give us as the minimum value for , which indeed is the answer posted above.
Problem
What is the product of the real roots of
the equation ?
Solution
If we expand by squaring, we get a quartic polynomial, which isn't very helpful.

Instead, we substitute for and our equation becomes .


Now we can square; solving for , we get or . The second solution is
extraneous since is positive. So, we have as the only solution for .
Substituting back in for ,

By Vieta's formulas, the product of the roots is .


Problem
A machine shop cutting tool is in the shape of a notched circle, as shown. The radius of
the circle is cm, the length of is 6 cm, and that of is 2 cm.
The angle is a right angle. Find the square of the distance (in centimeters)
from to the center of the circle.

Solution
Solution 1

Because we are given a right angle, we look for ways to apply the Pythagorean
Theorem. Let the foot of the perpendicular from to be and let the foot of the
perpendicular from to the line be . Let and . We're trying to
find .

Applying the Pythagorean Theorem, and .


Thus, , and . We solve this system to
get and , resulting in .

Problem
Suppose that the sum of the squares of two complex numbers and is and the sum
of the cubes is . What is the largest real value that can have?
Solution
Solution 1

One way to solve this problem seems to be by substitution.


and

Because we are only left with and , substitution won't be too bad.
Let and .
We get and
Because we want the largest possible , let's find an expression for in terms of .

.
Substituting, . Factored, (the
Rational Root Theorem may be used here, along with synthetic division)

The largest possible solution is therefore .


Solution 2

An alternate way to solve this is to let and .


Because we are looking for a value of that is real, we know that , and
thus .
Expanding will give two equations, since the real and imaginary parts
must match up.
Looking at the imaginary part of that equation, , so ,
and and are actually complex conjugates.
Looking at the real part of the equation and plugging in , ,
or .
Now, evaluating the real part of , which equals (ignoring the
odd powers of , since they would not result in something in the form of ):

Since we know that , it can be plugged in for in the above equation to


yield:

Since the problem is looking for to be a positive integer, only positive half-
integers (and whole-integers) need to be tested. From the Rational Roots

theorem, all fail, but does work. Thus, the real part of both
numbers is , and their sum is

Solution 3

Start by assuming x and y were roots of some polynomial of the form So

then and Substituting we arrive at the


polynomial From rational root theorem we find the roots to
be Since is the sum of the roots and is maximized when b is -4, the
answer is

Problem
Let equal . Determine the remainder upon dividing by .
Solution
Solution 1

First, we try to find a relationship between the numbers we're provided with and . We
realize that and both and are greater or less than by .
Expressing the numbers in terms of , we get .
Applying the Binomial Theorem, half of our terms cancel out and we are left
with . We realize that all of these terms are divisible
by except the final term.

After some quick division, our answer is .


Solution 2

Since (the Euler's totient function), by Euler's Totient


Theorem, where .

Thus

Alternatively, we could have noted that . This way, we


have , and can finish the same way.

Problem
Twenty five of King Arthur's knights are seated at their customary round table. Three of
them are chosen - all choices being equally likely - and are sent of to slay a
troublesome dragon. Let be the probability that at least two of the three had been
sitting next to each other. If is written as a fraction in lowest terms, what is the sum of
the numerator and the denominator?
Solution
Solution 1

We can use Complementary counting by finding the probability that none are sitting next
to each other and subtracting it from .
Imagine the other (indistinguishable) people are already seated, and fixed into place.
We will place , , and with and without the restriction.
There are places to place , followed by places to place , and places to
place after and . Hence, there are ways to place in between
these people with restrictions.
Without restrictions, there are places to place , followed by places to place ,
and places to place after and . Hence, there are ways to
place in between these people without restrictions.

Thus, the desired amount is , and the


answer is .
Solution 2

There are configurations for the knights about the table.

There are ways to pick a pair of knights from the trio, and there
are ways to determine which order they are seated. Since these two knights
must be attached, we let them be a single entity, so there
are configurations for the entities.
However, this overcounts the instances in which the trio sits together; when all three
knights sit together, then two of the pairs from the previous case are counted. However,
we only want to count this as one case, so we need to subtract the number of instances
in which the trio sits together (as a single entity). There are ways to determine
their order, and there are configurations.

Thus, the answer is , and the answer is .


Solution 3

Number the knights around the table 1-25. There are two possibilities: All three sit next
to each other, or two sit next to each other and one is not sitting next to the other two.
Case 1: All three sit next to each other. In this case, you are picking ,
, ... . This makes combinations.
Case 2: Like above, there are ways to pick the pair of knights sitting next to each
other. Once a pair is picked, you cannot pick either of the two adjacent knights. (i.e. if
you pick , you may not pick 4 or 7). Thus, there are ways to pick the
third knight, for a total of combinations.
Thus, you have a total of allowable ways to pick the knights.

The total number of ways to pick the knights is .

The probability is , and the answer is .


Solution 4

Pick an arbitrary spot for the first knight. Then pick spots for the next two knights in
order.
Case 1: The second knight sits next to the first knight. There are 2 possible places for

this out of 24, so the probability of this is . We do not need to consider the third
knight.
Case 2: The second knight sits two spaces from the first knight. There are 2 possible

places for this out of 24, so the probability is . Then there are 3 places out of a

remaining 23 for the third knight to sit, so the total probability for this case is
Case 3: The second knight sits 3 or more spaces from the first knight. There are 20

possible places for this out of 24, so the probability is . Then there are four places to

put the last knight out of 23, so the total probability for this case is
So add the probabilities to get the total:

Problem

What is the largest 2-digit prime factor of the integer ?


Solution
Expanding the binomial coefficient, we get . Let the prime be ;
then . If , then the factor of appears twice in the denominator.
Thus, we need to appear as a factor three times in the numerator, or . The
largest such prime is , which is our answer.
Problem

Find the minimum value of for .


Solution
Solution 1

Let . We can rewrite the expression as .


Since and because , we have . So we can apply AM-
GM:

The equality holds when .

Therefore, the minimum value is (when ; since is continuous

and increasing on the interval and its range on that interval is

from , by the Intermediate Value Theorem this value is attainable).


Solution 2

We can rewrite the numerator to be a perfect square by adding . Thus, we


must also add back .

This results in .
Thus, if , then the minimum is obviously . We show this possible
with the same methods in Solution 1; thus the answer is .
Solution 3

Let and rewrite the expression as , similar to the previous


solution. To minimize , take the derivative of and set it equal to zero.
The derivative of , using the Power Rule, is
=

is zero only when or . It can further be verified that and are


relative minima by finding the derivatives of other points near the critical points.

However, since is always positive in the given domain, .

Therefore, = , and the answer is .


Problem
The numbers , , and have something in common. Each is a four-digit
number beginning with that has exactly two identical digits. How many such numbers
are there?
Solution

Suppose the two identical digits are both one. Since the thousands digits must be one,
the other one can be in only one of three digits,

Because the number must have exactly two identical digits, , , and .
Hence, there are numbers of this form.
Suppose the two identical digits are not one. Therefore, consider the following
possibilities,

Again, , , and . There are numbers of this form.

Thus, the desired answer is


Problem
The solid shown has a square base of side length . The upper edge is parallel to the
base and has length . All other edges have length . Given that , what is the
volume of the solid?

Solution
Solution 1

First, we find the height of the figure by drawing a perpendicular from the midpoint
of to . The hypotenuse of the triangle is themedian of equilateral triangle ,
and one of the legs is . We apply the Pythagorean Theorem to find that the height
is equal to .

Next, we complete the figure into a triangular prism, and find the volume, which

is .
Now, we subtract off the two extra pyramids that we included, whose combined volume

is .

Thus, our answer is .


Problem
The length of diameter is a two digit integer. Reversing the digits gives the length of
a perpendicular chord . The distance from their intersection point to the
center is a positive rational number. Determine the length of .

Solution

Let and . It follows

that and . Applying the Pythagorean


Theorem on and

Because is a positive rational number, the quantity cannot


contain any square roots. Either or must be 11. However, cannot be
11, because both must be digits. Therefore, must equal eleven and must
be a perfect square (since ). The only pair that satisfies this
condition is , so our answer is .
Problem
For and each of its non-empty subsets, an alternating sum is defined as
follows. Arrange the number in the subset in decreasing order and then, beginning with
the largest, alternately add and subtract successive numbers. For example, the
alternating sum for is and for it is simply .
Find the sum of all such alternating sums for .
Solution 1

Let be a non- empty subset of .


Then the alternating sum of plus the alternating sum of with 7 included is 7. In
mathematical terms, . This is true because when we take an
alternating sum, each term of has the opposite sign of each corresponding term
of .
Because there are of these pairs, the sum of all possible subsets of our given set
is . However, we forgot to include the subset that only contains , so our answer
is .
Solution 2

Consider a given subset of that contains 7; then there is a subset which contains
all the elements of except for 7, and only those. Since each element of has one
element fewer preceding it than it does in , their signs are opposite; so the sum of the
alternating sums of and is equal to 7. There are subsets containing 7, so our
answer is .
Problem
In the adjoining figure, two circles with radii and are drawn with their centers units
apart. At , one of the points of intersection, a line is drawn in such a way that the
chords and have equal length. Find the square of the length of .

Solution
Solution 1

First, notice that if we reflect over we get . Since we know that is


on circle and is on circle , we can reflect circle over to get another circle
(centered at a new point with radius ) that intersects circle at . The rest is just
finding lengths:
Since is the midpoint of segment , is a median of triangle . Because we
know that , , and , we can find the third side of the
triangle using Stewart's Theorem or similar approaches. We get . So now
we have a kite with , , and , and all
we need is the length of the other diagonal . The easiest way it can be found is with
the Pythagorean Theorem. Let be the length of . Then

Doing routine algebra on the above equation, we find that ,


so
Solution 2

Draw additional lines as indicated. Note that since triangles and are
isosceles, the altitudes are also bisectors, so let .

Since triangles and are similar. If we let , we


have .
Applying the Pythagorean Theorem on triangle , we have .
Similarly, for triangle , we have .

Subtracting, .
Solution 3
Let . Angles , , and must add up to . By the Law
of Cosines, . Also,
angles and equal and . So we have

Taking the of both sides and simplifying using the cosine addition identity
gives .

Problem
The adjoining figure shows two intersecting chords in a circle, with on minor arc .
Suppose that the radius of the circle is , that , and that is bisected by .
Suppose further that is the only chord starting at which is bisected by . It
follows that the sine of the minor arc is a rational number. If this fraction is

expressed as a fraction in lowest terms, what is the product ?

Solution

Let be any fixed point on circle and let be a chord of circle .


The locus of midpoints of the chord is a circle , with diameter . Generally,
the circle can intersect the chord at two points, one point, or they may not have a
point of intersection. By the problem condition, however, the circle is tangent to BC at
point N.
Let M be the midpoint of the chord . From right triangle

, . Thus, .
Notice that the
distance equals (Where is the
radius of circle P). Evaluating
this, . From , we see

that
Next, notice that . We can therefore apply the tangent
subtraction formula to obtain

, . It follows

that , resulting in an answer of .

You might also like